harmonic series - generating function











up vote
3
down vote

favorite












I am currently learning about generating functions and
I found an interesting one for harmonic series, $dfrac{log(1-x)}{x-1}$.



Is there any hope I could get a formula for $n$th coefficient out of this? The $n$th derivative looks messy...



In absence of formula, can I at least get some asymptotic information, like that harmonic series diverges? (Can be shown more simply, I know.)










share|cite|improve this question




























    up vote
    3
    down vote

    favorite












    I am currently learning about generating functions and
    I found an interesting one for harmonic series, $dfrac{log(1-x)}{x-1}$.



    Is there any hope I could get a formula for $n$th coefficient out of this? The $n$th derivative looks messy...



    In absence of formula, can I at least get some asymptotic information, like that harmonic series diverges? (Can be shown more simply, I know.)










    share|cite|improve this question


























      up vote
      3
      down vote

      favorite









      up vote
      3
      down vote

      favorite











      I am currently learning about generating functions and
      I found an interesting one for harmonic series, $dfrac{log(1-x)}{x-1}$.



      Is there any hope I could get a formula for $n$th coefficient out of this? The $n$th derivative looks messy...



      In absence of formula, can I at least get some asymptotic information, like that harmonic series diverges? (Can be shown more simply, I know.)










      share|cite|improve this question















      I am currently learning about generating functions and
      I found an interesting one for harmonic series, $dfrac{log(1-x)}{x-1}$.



      Is there any hope I could get a formula for $n$th coefficient out of this? The $n$th derivative looks messy...



      In absence of formula, can I at least get some asymptotic information, like that harmonic series diverges? (Can be shown more simply, I know.)







      generating-functions






      share|cite|improve this question















      share|cite|improve this question













      share|cite|improve this question




      share|cite|improve this question








      edited Oct 28 '16 at 17:08









      Michael Hardy

      1




      1










      asked Oct 28 '16 at 16:06









      Adam

      1,45411537




      1,45411537






















          2 Answers
          2






          active

          oldest

          votes

















          up vote
          2
          down vote



          accepted










          $newcommand{bbx}[1]{,bbox[8px,border:1px groove navy]{{#1}},}
          newcommand{braces}[1]{leftlbrace,{#1},rightrbrace}
          newcommand{bracks}[1]{leftlbrack,{#1},rightrbrack}
          newcommand{dd}{mathrm{d}}
          newcommand{ds}[1]{displaystyle{#1}}
          newcommand{expo}[1]{,mathrm{e}^{#1},}
          newcommand{ic}{mathrm{i}}
          newcommand{mc}[1]{mathcal{#1}}
          newcommand{mrm}[1]{mathrm{#1}}
          newcommand{pars}[1]{left(,{#1},right)}
          newcommand{partiald}[3]{frac{partial^{#1} #2}{partial #3^{#1}}}
          newcommand{root}[2]{,sqrt[#1]{,{#2},},}
          newcommand{totald}[3]{frac{mathrm{d}^{#1} #2}{mathrm{d} #3^{#1}}}
          newcommand{verts}[1]{leftvert,{#1},rightvert}$

          With the identity
          $ds{pars{1 - x}^{m} = sum_{k = 0}^{infty}
          {m choose k}pars{-x}^{k} =
          sum_{k = 0}^{infty}{k - m - 1 choose k}x^{k}}$
          :




          begin{equation}
          begin{array}{l}
          mbox{Derivative respect of} ds{m}:
          \
          ds{pars{1 - x}^{m}lnpars{1 - x} =
          sum_{k = 0}^{infty}bracks{partiald{}{m}{k - m - 1 choose k}}x^{k}}
          \[5mm]
          mbox{The limit}ds{ m to - 1}:
          \
          ds{-,{lnpars{1 - x} over 1 - x} =
          sum_{k = 0}^{infty}color{#f00}{bracks{-,partiald{}{m}{k - m - 1 choose k}}
          _{ m = - 1}} x^{k}}
          end{array}
          label{1}tag{1}
          end{equation}



          begin{align}
          &color{#f00}{bracks{-,partiald{}{m}{k - m - 1 choose k}}_{ m = - 1}} =
          left.vphantom{Huge A}-,partiald{}{m}bracks{Gammapars{k - m} over k!,Gammapars{-m}}rightvert_{ m = - 1}
          \[5mm] = &
          -,{1 over k!},
          {-Gamma, 'pars{k + 1}Gammapars{1} + Gamma, 'pars{1}Gammapars{k + 1}over Gamma^{2}pars{1}}
          \[5mm] = &
          -,{1 over k!}bracks{%
          {-Gammapars{k + 1}Psipars{k + 1} +
          Gammapars{1}Psipars{1}Gammapars{k + 1}}}
          \[5mm] = &
          Psipars{k + 1} - Psipars{1} = color{#f00}{H_{k}}
          \[1cm]
          stackrel{mbox{see} eqref{1}}{implies} & ,,,
          bbox[10px,#ffe,border:1px dotted navy]{ds{%
          -,{lnpars{1 - x} over 1 - x} = sum_{k = 1}^{infty}H_{k}, x^{k}}}
          end{align}





          share|cite|improve this answer






























            up vote
            3
            down vote













            It is useful to notice that multiplication of a series $A(x)=sum_{n=0}^infty a_nx^n$ with the geometric series $frac{1}{1-x}$ transforms the sequence $(a_n)_{ngeq 0}$ to a sequence of sums $left(sum_{k=0}^na_kright)_{ngeq 0}$. We obtain



            begin{align*}
            frac{1}{1-x}A(x)&=frac{1}{1-x}sum_{n=0}^infty a_n x^n\
            &=sum_{n=0}^infty left(sum_{k=0}^n a_kright) x^ntag{1}
            end{align*}



            So, it is sufficient to determine the $n$-th coefficient of $A(x)$ in order to also know the $n$-th coefficient of $frac{1}{1-x}A(x)$.




            Since the series expansion of $-log (1-x)$ is known to be
            begin{align*}
            -log(1-x)=sum_{n=1}^infty frac{x^n}{n}qquadqquadqquad |x|<1
            end{align*}
            it follows from (1)
            begin{align*}
            -frac{log(1-x)}{1-x}&=frac{1}{1-x}sum_{n=1}^infty frac{x^n}{n}
            =sum_{n=1}^infty left(sum_{k=1}^nfrac{1}{k}right)x^n\
            &=sum_{n=1}^infty H_nx^n
            end{align*}







            share|cite|improve this answer























            • I probably wrote the question wrong. I actually started off wanting to sum 1+1/2+1/3+... - so I tried generating functions. By formula for nth coeffiecient I meant something like n*(n+1)/2 for 1+2+3+4+...n
              – Adam
              Oct 30 '16 at 5:59











            Your Answer





            StackExchange.ifUsing("editor", function () {
            return StackExchange.using("mathjaxEditing", function () {
            StackExchange.MarkdownEditor.creationCallbacks.add(function (editor, postfix) {
            StackExchange.mathjaxEditing.prepareWmdForMathJax(editor, postfix, [["$", "$"], ["\\(","\\)"]]);
            });
            });
            }, "mathjax-editing");

            StackExchange.ready(function() {
            var channelOptions = {
            tags: "".split(" "),
            id: "69"
            };
            initTagRenderer("".split(" "), "".split(" "), channelOptions);

            StackExchange.using("externalEditor", function() {
            // Have to fire editor after snippets, if snippets enabled
            if (StackExchange.settings.snippets.snippetsEnabled) {
            StackExchange.using("snippets", function() {
            createEditor();
            });
            }
            else {
            createEditor();
            }
            });

            function createEditor() {
            StackExchange.prepareEditor({
            heartbeatType: 'answer',
            convertImagesToLinks: true,
            noModals: true,
            showLowRepImageUploadWarning: true,
            reputationToPostImages: 10,
            bindNavPrevention: true,
            postfix: "",
            imageUploader: {
            brandingHtml: "Powered by u003ca class="icon-imgur-white" href="https://imgur.com/"u003eu003c/au003e",
            contentPolicyHtml: "User contributions licensed under u003ca href="https://creativecommons.org/licenses/by-sa/3.0/"u003ecc by-sa 3.0 with attribution requiredu003c/au003e u003ca href="https://stackoverflow.com/legal/content-policy"u003e(content policy)u003c/au003e",
            allowUrls: true
            },
            noCode: true, onDemand: true,
            discardSelector: ".discard-answer"
            ,immediatelyShowMarkdownHelp:true
            });


            }
            });














             

            draft saved


            draft discarded


















            StackExchange.ready(
            function () {
            StackExchange.openid.initPostLogin('.new-post-login', 'https%3a%2f%2fmath.stackexchange.com%2fquestions%2f1989255%2fharmonic-series-generating-function%23new-answer', 'question_page');
            }
            );

            Post as a guest















            Required, but never shown

























            2 Answers
            2






            active

            oldest

            votes








            2 Answers
            2






            active

            oldest

            votes









            active

            oldest

            votes






            active

            oldest

            votes








            up vote
            2
            down vote



            accepted










            $newcommand{bbx}[1]{,bbox[8px,border:1px groove navy]{{#1}},}
            newcommand{braces}[1]{leftlbrace,{#1},rightrbrace}
            newcommand{bracks}[1]{leftlbrack,{#1},rightrbrack}
            newcommand{dd}{mathrm{d}}
            newcommand{ds}[1]{displaystyle{#1}}
            newcommand{expo}[1]{,mathrm{e}^{#1},}
            newcommand{ic}{mathrm{i}}
            newcommand{mc}[1]{mathcal{#1}}
            newcommand{mrm}[1]{mathrm{#1}}
            newcommand{pars}[1]{left(,{#1},right)}
            newcommand{partiald}[3]{frac{partial^{#1} #2}{partial #3^{#1}}}
            newcommand{root}[2]{,sqrt[#1]{,{#2},},}
            newcommand{totald}[3]{frac{mathrm{d}^{#1} #2}{mathrm{d} #3^{#1}}}
            newcommand{verts}[1]{leftvert,{#1},rightvert}$

            With the identity
            $ds{pars{1 - x}^{m} = sum_{k = 0}^{infty}
            {m choose k}pars{-x}^{k} =
            sum_{k = 0}^{infty}{k - m - 1 choose k}x^{k}}$
            :




            begin{equation}
            begin{array}{l}
            mbox{Derivative respect of} ds{m}:
            \
            ds{pars{1 - x}^{m}lnpars{1 - x} =
            sum_{k = 0}^{infty}bracks{partiald{}{m}{k - m - 1 choose k}}x^{k}}
            \[5mm]
            mbox{The limit}ds{ m to - 1}:
            \
            ds{-,{lnpars{1 - x} over 1 - x} =
            sum_{k = 0}^{infty}color{#f00}{bracks{-,partiald{}{m}{k - m - 1 choose k}}
            _{ m = - 1}} x^{k}}
            end{array}
            label{1}tag{1}
            end{equation}



            begin{align}
            &color{#f00}{bracks{-,partiald{}{m}{k - m - 1 choose k}}_{ m = - 1}} =
            left.vphantom{Huge A}-,partiald{}{m}bracks{Gammapars{k - m} over k!,Gammapars{-m}}rightvert_{ m = - 1}
            \[5mm] = &
            -,{1 over k!},
            {-Gamma, 'pars{k + 1}Gammapars{1} + Gamma, 'pars{1}Gammapars{k + 1}over Gamma^{2}pars{1}}
            \[5mm] = &
            -,{1 over k!}bracks{%
            {-Gammapars{k + 1}Psipars{k + 1} +
            Gammapars{1}Psipars{1}Gammapars{k + 1}}}
            \[5mm] = &
            Psipars{k + 1} - Psipars{1} = color{#f00}{H_{k}}
            \[1cm]
            stackrel{mbox{see} eqref{1}}{implies} & ,,,
            bbox[10px,#ffe,border:1px dotted navy]{ds{%
            -,{lnpars{1 - x} over 1 - x} = sum_{k = 1}^{infty}H_{k}, x^{k}}}
            end{align}





            share|cite|improve this answer



























              up vote
              2
              down vote



              accepted










              $newcommand{bbx}[1]{,bbox[8px,border:1px groove navy]{{#1}},}
              newcommand{braces}[1]{leftlbrace,{#1},rightrbrace}
              newcommand{bracks}[1]{leftlbrack,{#1},rightrbrack}
              newcommand{dd}{mathrm{d}}
              newcommand{ds}[1]{displaystyle{#1}}
              newcommand{expo}[1]{,mathrm{e}^{#1},}
              newcommand{ic}{mathrm{i}}
              newcommand{mc}[1]{mathcal{#1}}
              newcommand{mrm}[1]{mathrm{#1}}
              newcommand{pars}[1]{left(,{#1},right)}
              newcommand{partiald}[3]{frac{partial^{#1} #2}{partial #3^{#1}}}
              newcommand{root}[2]{,sqrt[#1]{,{#2},},}
              newcommand{totald}[3]{frac{mathrm{d}^{#1} #2}{mathrm{d} #3^{#1}}}
              newcommand{verts}[1]{leftvert,{#1},rightvert}$

              With the identity
              $ds{pars{1 - x}^{m} = sum_{k = 0}^{infty}
              {m choose k}pars{-x}^{k} =
              sum_{k = 0}^{infty}{k - m - 1 choose k}x^{k}}$
              :




              begin{equation}
              begin{array}{l}
              mbox{Derivative respect of} ds{m}:
              \
              ds{pars{1 - x}^{m}lnpars{1 - x} =
              sum_{k = 0}^{infty}bracks{partiald{}{m}{k - m - 1 choose k}}x^{k}}
              \[5mm]
              mbox{The limit}ds{ m to - 1}:
              \
              ds{-,{lnpars{1 - x} over 1 - x} =
              sum_{k = 0}^{infty}color{#f00}{bracks{-,partiald{}{m}{k - m - 1 choose k}}
              _{ m = - 1}} x^{k}}
              end{array}
              label{1}tag{1}
              end{equation}



              begin{align}
              &color{#f00}{bracks{-,partiald{}{m}{k - m - 1 choose k}}_{ m = - 1}} =
              left.vphantom{Huge A}-,partiald{}{m}bracks{Gammapars{k - m} over k!,Gammapars{-m}}rightvert_{ m = - 1}
              \[5mm] = &
              -,{1 over k!},
              {-Gamma, 'pars{k + 1}Gammapars{1} + Gamma, 'pars{1}Gammapars{k + 1}over Gamma^{2}pars{1}}
              \[5mm] = &
              -,{1 over k!}bracks{%
              {-Gammapars{k + 1}Psipars{k + 1} +
              Gammapars{1}Psipars{1}Gammapars{k + 1}}}
              \[5mm] = &
              Psipars{k + 1} - Psipars{1} = color{#f00}{H_{k}}
              \[1cm]
              stackrel{mbox{see} eqref{1}}{implies} & ,,,
              bbox[10px,#ffe,border:1px dotted navy]{ds{%
              -,{lnpars{1 - x} over 1 - x} = sum_{k = 1}^{infty}H_{k}, x^{k}}}
              end{align}





              share|cite|improve this answer

























                up vote
                2
                down vote



                accepted







                up vote
                2
                down vote



                accepted






                $newcommand{bbx}[1]{,bbox[8px,border:1px groove navy]{{#1}},}
                newcommand{braces}[1]{leftlbrace,{#1},rightrbrace}
                newcommand{bracks}[1]{leftlbrack,{#1},rightrbrack}
                newcommand{dd}{mathrm{d}}
                newcommand{ds}[1]{displaystyle{#1}}
                newcommand{expo}[1]{,mathrm{e}^{#1},}
                newcommand{ic}{mathrm{i}}
                newcommand{mc}[1]{mathcal{#1}}
                newcommand{mrm}[1]{mathrm{#1}}
                newcommand{pars}[1]{left(,{#1},right)}
                newcommand{partiald}[3]{frac{partial^{#1} #2}{partial #3^{#1}}}
                newcommand{root}[2]{,sqrt[#1]{,{#2},},}
                newcommand{totald}[3]{frac{mathrm{d}^{#1} #2}{mathrm{d} #3^{#1}}}
                newcommand{verts}[1]{leftvert,{#1},rightvert}$

                With the identity
                $ds{pars{1 - x}^{m} = sum_{k = 0}^{infty}
                {m choose k}pars{-x}^{k} =
                sum_{k = 0}^{infty}{k - m - 1 choose k}x^{k}}$
                :




                begin{equation}
                begin{array}{l}
                mbox{Derivative respect of} ds{m}:
                \
                ds{pars{1 - x}^{m}lnpars{1 - x} =
                sum_{k = 0}^{infty}bracks{partiald{}{m}{k - m - 1 choose k}}x^{k}}
                \[5mm]
                mbox{The limit}ds{ m to - 1}:
                \
                ds{-,{lnpars{1 - x} over 1 - x} =
                sum_{k = 0}^{infty}color{#f00}{bracks{-,partiald{}{m}{k - m - 1 choose k}}
                _{ m = - 1}} x^{k}}
                end{array}
                label{1}tag{1}
                end{equation}



                begin{align}
                &color{#f00}{bracks{-,partiald{}{m}{k - m - 1 choose k}}_{ m = - 1}} =
                left.vphantom{Huge A}-,partiald{}{m}bracks{Gammapars{k - m} over k!,Gammapars{-m}}rightvert_{ m = - 1}
                \[5mm] = &
                -,{1 over k!},
                {-Gamma, 'pars{k + 1}Gammapars{1} + Gamma, 'pars{1}Gammapars{k + 1}over Gamma^{2}pars{1}}
                \[5mm] = &
                -,{1 over k!}bracks{%
                {-Gammapars{k + 1}Psipars{k + 1} +
                Gammapars{1}Psipars{1}Gammapars{k + 1}}}
                \[5mm] = &
                Psipars{k + 1} - Psipars{1} = color{#f00}{H_{k}}
                \[1cm]
                stackrel{mbox{see} eqref{1}}{implies} & ,,,
                bbox[10px,#ffe,border:1px dotted navy]{ds{%
                -,{lnpars{1 - x} over 1 - x} = sum_{k = 1}^{infty}H_{k}, x^{k}}}
                end{align}





                share|cite|improve this answer














                $newcommand{bbx}[1]{,bbox[8px,border:1px groove navy]{{#1}},}
                newcommand{braces}[1]{leftlbrace,{#1},rightrbrace}
                newcommand{bracks}[1]{leftlbrack,{#1},rightrbrack}
                newcommand{dd}{mathrm{d}}
                newcommand{ds}[1]{displaystyle{#1}}
                newcommand{expo}[1]{,mathrm{e}^{#1},}
                newcommand{ic}{mathrm{i}}
                newcommand{mc}[1]{mathcal{#1}}
                newcommand{mrm}[1]{mathrm{#1}}
                newcommand{pars}[1]{left(,{#1},right)}
                newcommand{partiald}[3]{frac{partial^{#1} #2}{partial #3^{#1}}}
                newcommand{root}[2]{,sqrt[#1]{,{#2},},}
                newcommand{totald}[3]{frac{mathrm{d}^{#1} #2}{mathrm{d} #3^{#1}}}
                newcommand{verts}[1]{leftvert,{#1},rightvert}$

                With the identity
                $ds{pars{1 - x}^{m} = sum_{k = 0}^{infty}
                {m choose k}pars{-x}^{k} =
                sum_{k = 0}^{infty}{k - m - 1 choose k}x^{k}}$
                :




                begin{equation}
                begin{array}{l}
                mbox{Derivative respect of} ds{m}:
                \
                ds{pars{1 - x}^{m}lnpars{1 - x} =
                sum_{k = 0}^{infty}bracks{partiald{}{m}{k - m - 1 choose k}}x^{k}}
                \[5mm]
                mbox{The limit}ds{ m to - 1}:
                \
                ds{-,{lnpars{1 - x} over 1 - x} =
                sum_{k = 0}^{infty}color{#f00}{bracks{-,partiald{}{m}{k - m - 1 choose k}}
                _{ m = - 1}} x^{k}}
                end{array}
                label{1}tag{1}
                end{equation}



                begin{align}
                &color{#f00}{bracks{-,partiald{}{m}{k - m - 1 choose k}}_{ m = - 1}} =
                left.vphantom{Huge A}-,partiald{}{m}bracks{Gammapars{k - m} over k!,Gammapars{-m}}rightvert_{ m = - 1}
                \[5mm] = &
                -,{1 over k!},
                {-Gamma, 'pars{k + 1}Gammapars{1} + Gamma, 'pars{1}Gammapars{k + 1}over Gamma^{2}pars{1}}
                \[5mm] = &
                -,{1 over k!}bracks{%
                {-Gammapars{k + 1}Psipars{k + 1} +
                Gammapars{1}Psipars{1}Gammapars{k + 1}}}
                \[5mm] = &
                Psipars{k + 1} - Psipars{1} = color{#f00}{H_{k}}
                \[1cm]
                stackrel{mbox{see} eqref{1}}{implies} & ,,,
                bbox[10px,#ffe,border:1px dotted navy]{ds{%
                -,{lnpars{1 - x} over 1 - x} = sum_{k = 1}^{infty}H_{k}, x^{k}}}
                end{align}






                share|cite|improve this answer














                share|cite|improve this answer



                share|cite|improve this answer








                edited Nov 12 at 18:20

























                answered Oct 30 '16 at 17:56









                Felix Marin

                65.8k7107138




                65.8k7107138






















                    up vote
                    3
                    down vote













                    It is useful to notice that multiplication of a series $A(x)=sum_{n=0}^infty a_nx^n$ with the geometric series $frac{1}{1-x}$ transforms the sequence $(a_n)_{ngeq 0}$ to a sequence of sums $left(sum_{k=0}^na_kright)_{ngeq 0}$. We obtain



                    begin{align*}
                    frac{1}{1-x}A(x)&=frac{1}{1-x}sum_{n=0}^infty a_n x^n\
                    &=sum_{n=0}^infty left(sum_{k=0}^n a_kright) x^ntag{1}
                    end{align*}



                    So, it is sufficient to determine the $n$-th coefficient of $A(x)$ in order to also know the $n$-th coefficient of $frac{1}{1-x}A(x)$.




                    Since the series expansion of $-log (1-x)$ is known to be
                    begin{align*}
                    -log(1-x)=sum_{n=1}^infty frac{x^n}{n}qquadqquadqquad |x|<1
                    end{align*}
                    it follows from (1)
                    begin{align*}
                    -frac{log(1-x)}{1-x}&=frac{1}{1-x}sum_{n=1}^infty frac{x^n}{n}
                    =sum_{n=1}^infty left(sum_{k=1}^nfrac{1}{k}right)x^n\
                    &=sum_{n=1}^infty H_nx^n
                    end{align*}







                    share|cite|improve this answer























                    • I probably wrote the question wrong. I actually started off wanting to sum 1+1/2+1/3+... - so I tried generating functions. By formula for nth coeffiecient I meant something like n*(n+1)/2 for 1+2+3+4+...n
                      – Adam
                      Oct 30 '16 at 5:59















                    up vote
                    3
                    down vote













                    It is useful to notice that multiplication of a series $A(x)=sum_{n=0}^infty a_nx^n$ with the geometric series $frac{1}{1-x}$ transforms the sequence $(a_n)_{ngeq 0}$ to a sequence of sums $left(sum_{k=0}^na_kright)_{ngeq 0}$. We obtain



                    begin{align*}
                    frac{1}{1-x}A(x)&=frac{1}{1-x}sum_{n=0}^infty a_n x^n\
                    &=sum_{n=0}^infty left(sum_{k=0}^n a_kright) x^ntag{1}
                    end{align*}



                    So, it is sufficient to determine the $n$-th coefficient of $A(x)$ in order to also know the $n$-th coefficient of $frac{1}{1-x}A(x)$.




                    Since the series expansion of $-log (1-x)$ is known to be
                    begin{align*}
                    -log(1-x)=sum_{n=1}^infty frac{x^n}{n}qquadqquadqquad |x|<1
                    end{align*}
                    it follows from (1)
                    begin{align*}
                    -frac{log(1-x)}{1-x}&=frac{1}{1-x}sum_{n=1}^infty frac{x^n}{n}
                    =sum_{n=1}^infty left(sum_{k=1}^nfrac{1}{k}right)x^n\
                    &=sum_{n=1}^infty H_nx^n
                    end{align*}







                    share|cite|improve this answer























                    • I probably wrote the question wrong. I actually started off wanting to sum 1+1/2+1/3+... - so I tried generating functions. By formula for nth coeffiecient I meant something like n*(n+1)/2 for 1+2+3+4+...n
                      – Adam
                      Oct 30 '16 at 5:59













                    up vote
                    3
                    down vote










                    up vote
                    3
                    down vote









                    It is useful to notice that multiplication of a series $A(x)=sum_{n=0}^infty a_nx^n$ with the geometric series $frac{1}{1-x}$ transforms the sequence $(a_n)_{ngeq 0}$ to a sequence of sums $left(sum_{k=0}^na_kright)_{ngeq 0}$. We obtain



                    begin{align*}
                    frac{1}{1-x}A(x)&=frac{1}{1-x}sum_{n=0}^infty a_n x^n\
                    &=sum_{n=0}^infty left(sum_{k=0}^n a_kright) x^ntag{1}
                    end{align*}



                    So, it is sufficient to determine the $n$-th coefficient of $A(x)$ in order to also know the $n$-th coefficient of $frac{1}{1-x}A(x)$.




                    Since the series expansion of $-log (1-x)$ is known to be
                    begin{align*}
                    -log(1-x)=sum_{n=1}^infty frac{x^n}{n}qquadqquadqquad |x|<1
                    end{align*}
                    it follows from (1)
                    begin{align*}
                    -frac{log(1-x)}{1-x}&=frac{1}{1-x}sum_{n=1}^infty frac{x^n}{n}
                    =sum_{n=1}^infty left(sum_{k=1}^nfrac{1}{k}right)x^n\
                    &=sum_{n=1}^infty H_nx^n
                    end{align*}







                    share|cite|improve this answer














                    It is useful to notice that multiplication of a series $A(x)=sum_{n=0}^infty a_nx^n$ with the geometric series $frac{1}{1-x}$ transforms the sequence $(a_n)_{ngeq 0}$ to a sequence of sums $left(sum_{k=0}^na_kright)_{ngeq 0}$. We obtain



                    begin{align*}
                    frac{1}{1-x}A(x)&=frac{1}{1-x}sum_{n=0}^infty a_n x^n\
                    &=sum_{n=0}^infty left(sum_{k=0}^n a_kright) x^ntag{1}
                    end{align*}



                    So, it is sufficient to determine the $n$-th coefficient of $A(x)$ in order to also know the $n$-th coefficient of $frac{1}{1-x}A(x)$.




                    Since the series expansion of $-log (1-x)$ is known to be
                    begin{align*}
                    -log(1-x)=sum_{n=1}^infty frac{x^n}{n}qquadqquadqquad |x|<1
                    end{align*}
                    it follows from (1)
                    begin{align*}
                    -frac{log(1-x)}{1-x}&=frac{1}{1-x}sum_{n=1}^infty frac{x^n}{n}
                    =sum_{n=1}^infty left(sum_{k=1}^nfrac{1}{k}right)x^n\
                    &=sum_{n=1}^infty H_nx^n
                    end{align*}








                    share|cite|improve this answer














                    share|cite|improve this answer



                    share|cite|improve this answer








                    edited Oct 29 '16 at 22:44

























                    answered Oct 29 '16 at 22:36









                    Markus Scheuer

                    58.9k454140




                    58.9k454140












                    • I probably wrote the question wrong. I actually started off wanting to sum 1+1/2+1/3+... - so I tried generating functions. By formula for nth coeffiecient I meant something like n*(n+1)/2 for 1+2+3+4+...n
                      – Adam
                      Oct 30 '16 at 5:59


















                    • I probably wrote the question wrong. I actually started off wanting to sum 1+1/2+1/3+... - so I tried generating functions. By formula for nth coeffiecient I meant something like n*(n+1)/2 for 1+2+3+4+...n
                      – Adam
                      Oct 30 '16 at 5:59
















                    I probably wrote the question wrong. I actually started off wanting to sum 1+1/2+1/3+... - so I tried generating functions. By formula for nth coeffiecient I meant something like n*(n+1)/2 for 1+2+3+4+...n
                    – Adam
                    Oct 30 '16 at 5:59




                    I probably wrote the question wrong. I actually started off wanting to sum 1+1/2+1/3+... - so I tried generating functions. By formula for nth coeffiecient I meant something like n*(n+1)/2 for 1+2+3+4+...n
                    – Adam
                    Oct 30 '16 at 5:59


















                     

                    draft saved


                    draft discarded



















































                     


                    draft saved


                    draft discarded














                    StackExchange.ready(
                    function () {
                    StackExchange.openid.initPostLogin('.new-post-login', 'https%3a%2f%2fmath.stackexchange.com%2fquestions%2f1989255%2fharmonic-series-generating-function%23new-answer', 'question_page');
                    }
                    );

                    Post as a guest















                    Required, but never shown





















































                    Required, but never shown














                    Required, but never shown












                    Required, but never shown







                    Required, but never shown

































                    Required, but never shown














                    Required, but never shown












                    Required, but never shown







                    Required, but never shown







                    Popular posts from this blog

                    Biblatex bibliography style without URLs when DOI exists (in Overleaf with Zotero bibliography)

                    ComboBox Display Member on multiple fields

                    Is it possible to collect Nectar points via Trainline?